Inflation is running 1.1% per year when you deposit $5,000 in an account earning 4.4% compounded continuously. In constant dollars, how much money will you have 6 years from now?
a. $6439.80
b. $6097.01
c. $1169.51
d. $6510.64

Answers

Answer 1
I think answer should be d. Please give me brainlest let me know if it’s correct or not okay thanks bye
Answer 2

The value of the money including inflation after 6 years will be $6510.64.

What is inflation?

The rate at which prices grow over a specific time period is known as inflation. Inflation is often measured in broad terms, such as the general rise in prices or the rise in a nation's cost of living. But it may also be computed more precisely for some products, like food, or for services, like a haircut, for instance. In any situation, inflation refers to how much more costly the pertinent collection of products and/or services has grown over a predetermined time frame, most frequently a year.

Given that, Inflation is running at 1.1% per year when you deposit $5,000 in an account earning 4.4% compounded continuously.

deposit amount (P) = $ 5000

Rate (r) = 4.4 %

Time (t) = 6 years

Compound continuously

Amount (A)= 5000 e⁰°⁰⁴⁴ˣ ⁶

A = $ 6,510.64098

A = $ 6510.64

Total accumulated money after 6 years = $ 6,510.64

value of money decreases due to inflation

The real value of money after n years with the rate of inflation i = 1.1%

yₙ = A( 1 / (1+i) )ⁿ

y₆ = 6510.64 (1 / ( 1 + 0.011))⁶

y₆ = 6,097.008

Real value of money = $ 6,097.01

Therefore, after 6 years real value of the money including inflation will be $ 6,097.01.

Learn more about inflation here:

https://brainly.com/question/29308595

#SPJ5


Related Questions

I’ll give brainlist to best answer

Answers

Answer:

12.25

Step-by-step explanation:

Expression

x^2 + 2y

x = 2.5

y = 3

x^2 = 6.25

2y = 3*2 = 6

x^2 + 2y = 6.25 + 6

x^2 + 2y = 12.25

Answer:

12.25

Step-by-step explanation:

We find the equation x^2 + 2y when x = 2.5 and y = 3.

We just have to replace x with 2.5 and y with 3 in the equation:

x^2 becomes 2.5^2

2y becomes 2 * 3 (When a variable is next to a number it means multiply)

So we get 2.5^2 + 2 * 3

We first do 2.5^2. 2.5^2 is the same as 2.5 * 2.5, which equals 6.25

Next we do 2 * 3 which equals 6

So it is 6.25 + 6, which is 12.25

Suppose that IQ scores have a bell-shaped distribution with a mean of 97 and a standard deviation of 17. Using the empirical rule, what percentage of IQ scores are between 46 and 148

Answers

Answer:

99.7% of IQ scores are between 46 and 148.

Step-by-step explanation:

The Empirical Rule states that, for a normally distributed random variable:

Approximately 68% of the measures are within 1 standard deviation of the mean.

Approximately 95% of the measures are within 2 standard deviations of the mean.

Approximately 99.7% of the measures are within 3 standard deviations of the mean.

In this problem, we have that:

Mean of 97, standard deviation of 17.

What percentage of IQ scores are between 46 and 148?

97 - 3*17 = 46

97 + 3*17 = 148

Within 3 standard deviations of the mean, so:

99.7% of IQ scores are between 46 and 148.

Listed below are the commissions earned ($000) last year by a sample of 15 sales representatives at Furniture Patch Inc.

$4.3 $6.1 $7.4 $10.7 $13.1 $13.6 $14.7 $16.5 $17.1 $17.4 $18.7 $22.3 $36.7 $43.2 $79.4

Required:
a. Determine the mean, median, and the standard deviation.
b. Determine the coefficient of skewness using Pearson.

Answers

Answer:

Mean = 21.413

Median = 16.5

Standard deviation = 19.182

0.768

Step-by-step explanation:

Given the data :

Ordered data, X : 4.3 6.1 7.4 10.7 13.1 13.6 14.7 16.5 17.1 17.4 18.7 22.3 36.7 43.2 79.4

The sample size, n = 15

The mean, m= ΣX / n = 321.2 / 15 = 21.413

The median, Md:

1/2(n + 1)th term

1/2 (15 +1) th term

1/2(16) = 8th term

Median = $16.5

The standard deviation, s = √[Σ(x - m)²/ n]

The standard deviation obtained using a calculator is ; 19.182

Coefficient of skewness : = 3(m - Md) / s

= 3(21.413 - 16.5) / 19.182

= 14.739 / 19.182

= 0.768

de moirve's
(√3-i ÷ √3+i)^6 = 1

Answers

(√3 - i ) / (√3 + i ) × (√3 - i ) / (√3 - i ) = (√3 - i )² / ((√3)² - i ²)

… = ((√3)² - 2√3 i + i ²) / (3 - i ²)

… = (3 - 2√3 i - 1) / (3 - (-1))

… = (2 - 2√3 i ) / 4

… = 1/2 - √3/2 i

… = √((1/2)² + (-√3/2)²) exp(i arctan((-√3/2)/(1/2))

… = exp(i arctan(-√3))

… = exp(-i arctan(√3))

… = exp(-/3)

By DeMoivre's theorem,

[(√3 - i ) / (√3 + i )]⁶ = exp(-6/3) = exp(-2) = 1

Use technology to help you test the claim about the population mean, mu, at the given level of significance, alpha, using the given sample statistics. Assume the population is normally distributed.
Claim: μ>1220;α=0.08; σ=211.67.
Sample statistics: x=1235.91,n=300
Identify the null and alternative hypotheses and calculate the standardized test statistic.

Answers

Answer:

H0 : μ = 1220

H1 : μ > 1220

Test statistic = 1.30

Step-by-step explanation:

Sample mean, x = 1235.91

Standard deviation, σ = 211.67

Sample size, n = 300

The hypothesis :

Null ; H0 : μ = 1220

Alternative ; H1 : μ > 1220

Tbe test statistic :

(x - μ) ÷ (σ/√(n))

(1235.91 - 1220) ÷ (211.67/√(300))

15.91 / 12.220773

= 1.3018

= 1.30

If you were given a fractional strip, that did not have any subdivisions marked like this one pictured below, how would you determine the fractional amount of the bar that is shaded?

Answers

9514 1404 393

Answer:

  it depends on the accuracy and resolution required of the answer

Step-by-step explanation:

The shaded portion appears to be about half the length of the unshaded portion, suggesting the shaded amount is 1/3.

__

Using a pair of dividers, one could determine the number of times the shaded portion fits into the whole bar. Depending on how much is left over, the process could repeat to determine the approximate size of the remaining fraction relative to the bar or to the shaded portion. (Alternatively, one could replicate the length of the bar to see what integer number of shaded lengths fit into what integer number of whole lengths.)

One could measure the shaded part and the whole bar with a ruler, then determine the relative size of the shaded part by dividing the first measurement by the second. The finer the divisions on the ruler, the better the approximation will be.

please help me to do I want primeter and area of this​

Answers

1. Perimeter- 6+6+3+3= 18cm
Area- 6*3=18 sq cm

2. Perimeter- 3+5+4=12cm
Area- 6 sq cm

3. Perimeter- 2+2+7+7= 18cm
Area- 2*7= 14 sq cm

4. Perimeter- 9+9+9+9= 36cm
Area- 9*9= 81 sq cm

An employer has a staff of eighty actuaries, ten of whom are student actuaries. A student actuary is allowed a total of ten weeks off per year (52 weeks in a year) for studying, vacation, and sick days. A non-student actuary is given four weeks off a year. It is assumed that all actuaries use all of the weeks off allocated to them. The actuary Mr. Taylor is at work today. What is the probability that he is a student?

Answers

Answer:

0.1111

Step-by-step explanation:

From the given information;

Number of staffs in the actuary = 80

Out of the 80, 10 are students.

i.e.

P(student actuary) = 10/80 = 0.125

number of weeks in a year = 52

off time per year = 10/52 = 0.1923

P(at work || student actuary) = (50 -10/52)

= 42/52

= 0.8077

P(non student actuary) = (80 -10)/80

= 70 / 80

= 0.875

For a non-student, they are only eligible to 4 weeks off in a year

i.e.

P(at work | non student) = (52-4)/52

= 48/52

= 0.9231

P(at work) = P(student actuary) × P(at work || student actuary) + P(non student actuary) × P(at work || non studnet actuary)

P(at work) =  (0.125 × 0.8077) + ( 0.875 × 0.9231)

P(at work) = 0.1009625 + 0.8077125

P(at work) = 0.90868

Finally, the P(he is a student) = (P(student actuary) × P(at work || student actuary) ) ÷ P(at work)

P(he is a student) = (0.125 × 0.8077) ÷ 0.90868

P(he is a student) = 0.1009625 ÷ 0.90868

P(he is a student) = 0.1111

i already have A but I do not have B

Answers

Answer:

-4 , -1 , -2 , 0 , +1 , +3

Step-by-step explanation:

Answer:

the integers -4,-2,-1,0, +1, +3

Step-by-step explanation:

because when you put them in order  you find which pairs are located between -5 and +5

-8,-4,-2,-1,0,+3,+8,+9

which tells you that

-4,-2,-1,0, +1, +3 are between -5 and +5

Solve for 5x + 11 ≤ 67 = ?

9I will give brainliest.)

Answers

Answer:

x ≤ 11.20

Step-by-step explanation:

solve it like a regular equation

5x ≤ 67 - 11

5x ≤ 56

x ≤ 11 1/5

x ≤ 11.20

A matinee ticket costs $6 per adult and $1 per child. On a certain day, the total number of adults (a) and children (c) who saw a movie was 35, and the total money collected was $70. Which of the following options represents the number of children and the number of adults who saw a movie that day, and the pair of equations that can be solved to find the numbers?

7 children and 28 adults
Equation 1: a + c = 35
Equation 2: 6a − c = 70
7 children and 28 adults
Equation 1: a + c = 35
Equation 2: 6a + c = 70
28 children and 7 adults
Equation 1: a + c = 35
Equation 2: 6a + c = 70
28 children and 7 adults
Equation 1: a + c = 35
Equation 2: 6a − c = 70

Answers

Answer:

28 children and 7 adults

Equation 1: a + c = 35

Equation 2: 6a + c = 70

Step-by-step explanation:

If the total number of people at the movie was 35 people, one of the equations will be a + c = 35.

If $70 was collected in total, the other equation will be 6a + c = 70.

Now, solve this system of equations:

a + c = 35

6a + c = 70

Solve by elimination by multiplying the top equation by -1, then adding the equations together:

-a - c = -35

6a + c = 70

Add these together, and solve for a:

5a = 35

a = 7

Since there were 35 people in total, find how many children attended by subtracting 7 from 35:

35 - 7

= 28

So, there were 28 children and 7 adults.

The equations used were: a + c = 35 and 6a + c = 70

So, the correct answer is:

28 children and 7 adults

Equation 1: a + c = 35

Equation 2: 6a + c = 70

Kyle buys a bag of cookies that contains 4 chocolate chip cookies, 9 peanut butter cookies, 9 sugar cookies and 7 oatmeal cookies. What is the probability that Kyle randomly selects a sugar cookie from the bag, eats it, then randomly selects a peanut butter cookie

Answers

Multiply all them then stubbtrac to number

Which graph shows the solution to this system of inequalities?
y>-1/3x+1
y>2x-3

Answers

Given:

The system of inequalities is:

[tex]y>-\dfrac{1}{3}x+1[/tex]

[tex]y>2x-3[/tex]

To find:

The graph of the given system of inequalities.

Solution:

We have,

[tex]y>-\dfrac{1}{3}x+1[/tex]

[tex]y>2x-3[/tex]

The related equations are:

[tex]y=-\dfrac{1}{3}x+1[/tex]

[tex]y=2x-3[/tex]

Table of values for the given equations is:

    [tex]x[/tex]                   [tex]y=-\dfrac{1}{3}x+1[/tex]             [tex]y=2x-3[/tex]

   0                             1                              -3

   3                             0                              3

Plot (0,1) and (3,0) and connect them by a straight line to get the graph of [tex]y=-\dfrac{1}{3}x+1[/tex].

Similarly, plot (0,-3) and (3,3) and connect them by a straight line to get the graph of [tex]y=2x-3[/tex].

The signs of both inequalities are ">". So, the boundary line is a dotted line and the shaded region or each inequality lie above the boundary line.

Therefore, the graph of the given system of inequalities is shown below.

20. Find the measure of < DEG. (G.CO.C.10)
4
E
A. 25
B. 8
(3y + 4) A (5y-10)
C. 30

D
F
Click to add speaker notes
e here to search
O
c
3
PO
.
a

Answers

Answer:

A. 25

Step-by-step explanation:

From the diagram given, we can deduce that <D EG = <F EG

Therefore:

3y + 4 = 5y - 10

Collect like terms and solve for y

3y - 5y = -4 - 10

-2y = -14

Divide both sides by -2

y = -14/-2

y = 7

✔️m<D EG = 3y + 4

Plug in the value of y

m<D EG = 3(7) + 4

m<D EG = 25°

A.85
B.98
C.102
D.34

Answers

Answer:

C. 102

Step-by-step explanation:

[tex]{hope it helps}}[/tex]

How would this quadrilateral be best classified, and what is the measure of Angle B?

Answers

Answer:

The quadrilateral is Rhombus

B=70°

Step-by-step explanation:

110+110+z+z=360

220+2z=360

2z=360-220

2z=140

z=140/2

Therefore, z=70

So Angle B=70

Since z= Angle B=Angle D

A money box contains only 10-cent
and 20-cent coins. There are 33
coins with a total value of $4.60.
How many coins of each?

Answers

Answer:

Below in bold.

Step-by-step explanation:

x + y = 33      where x = 10 cent coin and y = 20 cent coin

10x + 20y = 460

Multiply first equation by 10:

10x + 10y = 330

Subtract this from the second equation:

10y = 130

y = 13

So there are 13 20c coins

and 33 - 13 = 20 10c coins.

20 ten cents and 13 twenty cents
This is a real logic problem you just need to trial and error

A rectangular storage container with an open top is to have a volume of 14 cubic meters. The length of its base is twice the width. Material for the base costs 10 dollars per square meter. Material for the sides costs 8 dollars per square meter. Find the cost of materials for the cheapest such container.

Answers

Answer:

C(min)  =  277.95 $

Container dimensions:

x = 2.822 m

y = 1.411 m

h = 3.52 m

Step-by-step explanation:

Let´s call x  and  y the sides of the rectangular base.

The surface area for  a rectangular container is:

S = Area of the base (A₁) +  2 * area of a lateral side x (A₂) + 2 * area lateral y (A₃)

Area of the base is :

A₁ =  x*y       we assume, according to problem statement that

x  =  2*y      y  = x/2

A₁  =  x²/2

Area lateral on side x

A₂  = x*h           ( h  is the height of the box )

Area lateral on side y

A₃ = y*h        ( h  is the height of the box )

s = x²/2  + 2*x*h + 2*y*h

Cost  =  Cost of the base + cost of area lateral on x + cost of area lateral on y

C = 10*x²/2  + 8* 2*x*h  + 8*2*y*h

C as function of x is:

The volume of the box is:

V(b) = 14 m³  = (x²/2)*h       28 = x²h      h = 28/x²

C(x) = 10*x²/2  + 16*x*28/x² + 16*(x/2)*28/x²

C(x)  =  5*x²  +  448/x  + 224/x

Taking derivatives on both sides of the equation we get:

C´(x)  =  10*x  -  448/x² - 224/x²

C´(x)  =  0             10x  -  448/x² - 224/x² = 0     ( 10*x³ - 448 - 224 )/x² = 0

10*x³ - 448 - 224 = 0       10*x³ = 224

x³  =22.4

x = ∛ 22.4

x = 2.822 m

y = x/2  =  1.411 m

h = 28/x²  =  28 /7.96

h =  3.52 m

To find out if the container of such dimension is the cheapest container we look to the second derivative of C

C´´(x) = 10 + 224*2*x/x⁴

C´´(x)  =  10 + 448/x³    is positive then C has a minimum for x = 2.82

And the cost of the container is:

C = 10*(x²/2) +  16*x*h + 16*y*h

C = 39.82 +  158.75  + 79.38

C =  277.95 $

if $1995 .00 is Shared equally among 7 men, how much would each get?​

Answers

Anwer:$285

Explaination: Division method

$1995.00÷7=$285

Pls help I don’t understand this one pls

Answers

Answer:

15=225

20= 400

Step-by-step explanation:

the small 2 means multiply that number two times :)

Answer:

[tex]15^2\\[/tex] = 225

[tex]20^{2}[/tex] = 400

Step-by-step explanation:

[tex]x^{2}[/tex] = [tex]x[/tex] × [tex]x[/tex]

basically it's the number times itself

15 × 15 = 225

20 × 20 = 400

Does anyone know the answer to this? Algebra 2
I have to find the answers to
Find cos 0
Find tan 0
Find csc 0
Find sec 0
Find cot 0
And what terminal of the angle falls in which quadrant? 1-4?

Answers

Answer:

Step-by-step explanation:

Sinθ = [tex]\frac{\text{Opposite side}}{\text{Hypotenuse}}[/tex]

If, sinθ = -[tex]\frac{1}{2}[/tex] and π < θ < [tex]\frac{3\pi }{2}[/tex]

Since, sinθ is negative, angle θ will be in IIIrd quadrant.

And the measure of angle θ will be (180° + 30°)

θ = 210°

It's necessary to remember that tangent and cotangent of angle θ in quadrant III are positive.

Therefore, cos(210°) = [tex]-\frac{\sqrt{3} }{2}[/tex]

tan(210°) = [tex]\frac{1}{\sqrt{3} }[/tex]

csc(210°) = [tex]-\frac{1}{2}[/tex]

sec(210°) = [tex]-\frac{2}{\sqrt{3} }[/tex]

cot(210°) = √3

Find the constant of variation when t varies directly as s, and t =
260 when s = 65.

Answers

Answer:

4

Step-by-step explanation:

Use the direct variation equation, y = kx.

Replace y with t, and replace x with s:

y = kx

t = ks

Plug in 260 as k and 65 as s, then solve for k (the constant of variation):

t = ks

260 = k(65)

4 = k

So, the constant of variation is 4.

Find the length of the third side. If necessary, round to the nearest tenth

Answers

[tex]\huge\bold{Given:}[/tex]

Length of the base = 8

Length of the hypotenuse = 17

[tex]\huge\bold{To\:find:}[/tex]

The length of the third side ''[tex]x[/tex]".

[tex]\large\mathfrak{{\pmb{\underline{\orange{Solution}}{\orange{:}}}}}[/tex]

[tex]\longrightarrow{\purple{x\:=\: 15}}[/tex] 

[tex]\large\mathfrak{{\pmb{\underline{\red{Step-by-step\:explanation}}{\red{:}}}}}[/tex]

Using Pythagoras theorem, we have

(Perpendicular)² + (Base)² = (Hypotenuse)²

[tex]\longrightarrow{\blue{}}[/tex] [tex]{x}^{2}[/tex] + (8)² = (17)²

[tex]\longrightarrow{\blue{}}[/tex] [tex]{x}^{2}[/tex] + 64 = 289

[tex]\longrightarrow{\blue{}}[/tex]  [tex]{x}^{2}[/tex] = 289 - 64

[tex]\longrightarrow{\blue{}}[/tex]  [tex]{x}^{2}[/tex] = 225

[tex]\longrightarrow{\blue{}}[/tex]  [tex]x[/tex] = [tex]\sqrt{225}[/tex]

[tex]\longrightarrow{\blue{}}[/tex]  [tex]x[/tex] = [tex]15[/tex]

Therefore, the length of the missing side [tex]x[/tex] is [tex]15[/tex].

[tex]\huge\bold{To\:verify :}[/tex]

[tex]\longrightarrow{\green{}}[/tex] (15)² + (8)² = (17)²

[tex]\longrightarrow{\green{}}[/tex] 225 + 64 = 289

[tex]\longrightarrow{\green{}}[/tex] 289 = 289

[tex]\longrightarrow{\green{}}[/tex] L.H.S. = R. H. S.

Hence verified.

[tex]\huge{\textbf{\textsf{{\orange{My}}{\blue{st}}{\pink{iq}}{\purple{ue}}{\red{35}}{\green{♨}}}}}[/tex]

Workers employed in a large service industry have an average wage of $9.00 per hour with a standard deviation of $0.50. The industry has 64 workers of a certain ethnic group. These workers have an average wage of $8.85 per hour. Calculate the probability of obtaining a sample mean less than or equal to $8.85 per hour. (Round your answer to four decimal places.)

Answers

Answer:

The probability of obtaining a sample mean less than or equal to $8.85 per hour=0.0082

Step-by-step explanation:

We are given that

Average wage, [tex]\mu=[/tex]$9.00/hour

Standard deviation,[tex]\sigma=[/tex]$0.50

n=64

We have to find the  probability of obtaining a sample mean less than or equal to $8.85 per hour.

[tex]P(\bar{x} \leq 8.85)=P(Z\leq \frac{\bar{x}-\mu}{\frac{\sigma}{\sqrt{n}}})[/tex]

Using the values

[tex]P(\bar{x}\leq 8.85)=P(Z\leq \frac{8.85-9}{\frac{0.50}{\sqrt{64}}})[/tex]

[tex]P(\bar{x}\leq 8.85)=P(Z\leq \frac{-0.15}{\frac{0.50}{8}})[/tex]

[tex]P(\bar{x}\leq 8.85)=P(Z\leq -2.4)[/tex]

[tex]P(\bar{x}\leq 8.85)=0.0082[/tex]

Hence, the probability of obtaining a sample mean less than or equal to $8.85 per hour=0.0082

Razon trigonometría que se requiere para calcular la altura de la torre si desde una distancia de 50 m se observa su punto mas alto con un ángulo de 48

Answers

Answer:

se supone que debes usar el SINE RATIO ya que se trata del lado opuesto y la hipotenusa.

Crystal left her running shoes at school yesterday. Today she walked 44 miles to school to get her shoes, she ran home along the same route, and the total time for both trips was 22 hours. Crystal walked and ran at constant speeds, and she ran 33 miles per hour faster than she walked.

What was Crystal’s walking speed in miles per hour?

Answers

Answer:

We can conclude that her walking speed is 2.1 miles per hour.

Step-by-step explanation:

We have the relation:

Speed = distance/time.

Here we know:

She walked for 44 miles.

And she ran along the same route, so she ran for 44 miles.

The total time of travel is 22 hours, so if she ran for a time T, and she walked for a time T', we must have:

T + T' = 22 hours.

If we define: S = speed runing

                     S' = speed walking

Then we know that:

"and she ran 33 miles per hour faster than she walked."

Then:

S = S' + 33mi/h

Then we have four equations:

S'*T' = 44 mi

S*T = 44 mi

S = S' + 33mi/h

T + T' = 22 h

We want to find the value of S', the speed walking.

To solve this we should start by isolating one of the variables in one of the equations.

We can see that S is already isoalted in the third equation, so we can replace that in the other equations where we have the variable S, so now we will get:

S'*T' = 44mi

(S' + 33mi/H)*T = 44mi

T + T' = 22h

Now let's isolate another variable in one of the equations, for example we can isolate T in the third equation to get:

T = 22h - T'

if we replace that in the other equations we get:

S'*T' = 44mi

(S' + 33mi/h)*(  22h - T') = 44 mi

Now we can isolate T' in the first equation to get:

T' = 44mi/S'

And replace that in the other equation so we get:

(S' + 33mi/h)*(  22h -44mi/S' ) = 44 mi

Now we can solve this for S'

22h*S' + (33mi/h)*22h + S'*(-44mi/S')  + 33mi/h*(-44mi/S') = 44mi

22h*S' + 726mi - 44mi - (1,452 mi^2/h)/S' = 44mi

If we multiply both sides by S' we get:

22h*S'^2 + (726mi - 44mi)*S' - (1,425 mi^2/h) = 44mi*S'

We can simplify this to get:

22h*S'^2 + (726mi - 44mi - 44mi)*S' - (1,425 mi^2/h) = 0

22h*S'^2 + (628mi)*S' - ( 1,425 mi^2/h) = 0

This is just a quadratic equation, the solutions for S' are given by the Bhaskara's equation:

[tex]S' = \frac{-628mi \pm \sqrt{(628mi)^2 - 4*(22h)*(1,425 mi^2/h)} }{2*22h} \\S' = \frac{-628mi \pm 721 mi }{44h}[/tex]

Then the two solutions are:

S' = (-628mi - 721mi)/44h = -30.66 mi/h

But this is a negative speed, so this has no real meaning, and we can discard this solution.

The other solution is:

S' = (-628mi + 721mi)/44h = 2.1 mi/h

We can conclude that her walking speed is 2.1 miles per hour.

HCF of the numbers divisible be
3 between 21 and 30 is ___​

Answers

Answer:

3

Step-by-step explanation:

Numbers between 21 and 30 divisible by 3 are 24 and 27. so you get the HCF of the two.

Assume for a paired-samples t test: N= 17, Mdifference = 467.72, s = 264.50. What is the effect size statistic?

Answers

Answer:

bbbbbhbbvcgccfggfgggggggggihh

This question difficult and i need some help would anyone please help me

Answers

Answer:

x = 30

F = 130

G =  50

Step-by-step explanation:

f and g are supplementary which means they add to 180

5x-20 + 3x - 40 = 180

Combine like terms

8x - 60 = 180

Add 60 to each side

8x-60+60 = 180+60

8x = 240

Divide by 8

8x/8 = 240/8

x = 30

F = 5x -20 = 5*30 -20 = 150 -20 = 130

G = 3x-40 = 3*30 -40 = 90-40 = 50

Answer:

Because a straight line = 180, we can find x like this :

(5x - 20) + (3x - 40) = 180

Step 1 - collect like terms

8x - 60 = 180

Step 2 - Move terms around to isolate x

8x = 180 + 60

Step 3 - Divide both sides by 8

x = 30

Now you can find the value of the angles by plugging in x

∠f = (5 x 30) - 20

     = 130 degrees

∠g = (3 x 30) - 40

      = 50 degrees

We can check to see if this works by adding them up

130 + 50 = 180, so this is correct

Hope this helps! I would really appreciate a brainliest if possible :)

The diameter of the circle below is 82cm. Work out the radius of the circle

Answers

Answer:

Radius = 41

Step-by-step explanation:

Diameter/2=radius

82/2 =41

Answer:

Radius=41

Step-by-step explanation:

Preamble

Diameter=82

Radius=?

Formula

Radius=diameter/2

Radius=82/2

reduce the fraction

82/2=82÷2/2÷2=41/1

therefore radius=41

Other Questions
. A 79 g sample of water at 21oC is heated until it becomes steam with a temperature of 143oC. Find the change in heat content of the system. Select the answer that best describes why roger didnt run when he had the chance. Cu 29. Trong cc hm s sau, hm s no l hm s chn? A. 2cos sin 2 . 2 y x x B. sin sin . 4 4 y x x C. 2 sin sin . 4 y x x D. y x IAD B. WELLS WROTE ARTICLES TO The graph shows the value of a certain model of car compared with its age.Which statement is false?Car's value ($)Car's ago (years)A. The correlation coefficient is close to -1,B. The car's value is the explanatory variable.O O OdC. The car's age is strongly correlated to its value.D. The data show a negative linear relationship. what is the value n ? Find the area of the parallelogram in the figure below. Round your final answer to the nearest tenth.16.8D23.223.212.4B What are biofertilisiers In the diagram below of triangle PQS, S is the midpoint of PR and T is the midpoint of QR. If ST = 5x - 22, and PQ = 3x + 19, what is the measurement of PQ? Leila is arranging 11 cans of food in a row on a shelf. She has 4 cans of corn, 1 can of peas, and 6 cans of beets. In how many distinct orders can the cans be arranged if two cans of the same food are considered identical. The difference between the actual labor rate multiplied by the actual labor hours worked and the standard labor rate multiplied by the standard labor hours is the:_________. a. labor price variance. b. total labor variance. c. labor efficiency variance. d. labor quantity variance. Name a geometric figure that could be the height of a tree plsssssss ill also rate brainlist in which of the following type of colloid, the physical state of dispersed phase and dispersion medium is same? Gel Foam Emulsion Aerosol What is the answer for this one what is the mass of alumunium will be completly oxidized by 44.8 L of oxygen to produce Al2o3 at STP? Compared to a few decades ago, personality psychologists today are much more likely to acknowledge the importance of biological influences on personality. This shift is partly due to:___.a. the overwhelming influence of the trait model in personality psychology. b. the increase in researchers with MD's getting into the field of personality research.c. the increasing use of twin studies in all personality research improvements.d. in research instruments that allow researchers to assess personality and biology more accurately. plz help worth 50 points In the diagram, the lines dividing parking spaces are parallel. The measure of 1 is 110. Identify the measures of each labeled angle to ensure cars can park safely. Paraphrase this pls plllllllls i will mark as brainliest